<html><head><meta http-equiv="Content-Type" content="text/html charset=utf-8"></head><body style="word-wrap: break-word; -webkit-nbsp-mode: space; -webkit-line-break: after-white-space;" class=""><div class="">Hi Chip (and all), </div><div class="">   Thanks for summarizing your approach to calculating electron spin from your model.</div><div class="">You quoted Wikipedia:</div><div class=""><br class=""></div><div class=""> <span style="font-family: 'Times New Roman', serif; font-size: 12pt; background-color: rgb(255, 255, 255);" class="">Please see PHOTON in Wikipedia…</span><span style="font-family: 'Times New Roman', serif; font-size: 12pt; background-color: rgb(255, 255, 255);" class=""> </span><i class="" style="font-family: 'Times New Roman', serif; font-size: 12pt;">“The photon also carries a quantity called spin angular momentum that does not depend on its frequency. The magnitude of its spin is sqrt(2) hbar and the component measured along its direction of motion, its helicity, must be ±ħ.”</i></div><div class=""><i class="" style="font-family: 'Times New Roman', serif; font-size: 12pt;"><br class=""></i></div><div class="">I did vector calculations for the total angular momentum L= R x p of both my spin 1 hbar and spin 1/2 hbar transluminal energy quantum photon models. The results were as follows for L^2 , which is the square of the total angular momentum of the photon models.</div><div class=""><br class=""></div><div class="">For the spin 1 hbar photon model:     L^2 = 2 hbar^2  + (Et)^2  where E is the energy of the photon and t is the time (arbitrary starting time) for the mathematical helical motion of the energy quantum.</div><div class=""><br class=""></div><div class="">For the spin 1/2 hbar charged photon model:   L^2 = 2 (hbar/2)^2 + (Et)^2 for E and t as above.</div><div class=""><br class=""></div><div class="">This shows that as t -> 0  (remembering that the starting point of t is arbitrary in the models, the above total angular momentum equations for the photon models reduce to  L= sqrt(2) hbar  for the spin 1 photon model and L= sqrt(2) (hbar/2) for the spin 1/2 photon model. This is the quantum mechanical result for the total spin of a spin 1 photon (as mentioned above in the Wikipedia quote) , remembering that Sz = hbar  and hbar/2 are the z-components of spin of a photon or electron respectively that are actually measured in photon and electron experiments. I had  never previously done these calculations for the total spin of my spin 1 and spin 1/2 charged photon models (whose equations are summarized in my recent article at <a href="https://www.academia.edu/30899196/Transluminal_Energy_Quantum_Model_of_a_Spin-_Charged_Photon_Composing_an_Electron" class="">https://www.academia.edu/30899196/Transluminal_Energy_Quantum_Model_of_a_Spin-_Charged_Photon_Composing_an_Electron</a> ) which is attached below. So the fact that the result for the total spin sqrt(2) hbar of my spin 1 model photon is consistent with the quantum mechanical result for a spin 1 photon  is a very nice result. The expression (Et)^2 in the total angular momentum L^2 result is curious. But I think this part of the result is because in the calculation of L=R x p ,  R is measured from the origin (x=0, y=0, z=0) of the coordinate system in which the helical motion is given.  Since for large values of t,  R increases with time approximately as R=ct, while p = (E/c) sqrt(2) for the transluminal energy quantum of the photon models, the vector calculation of L= R x p =~  ct x (E/c) (sqrt (2)= Et is also increasing with time (which accounts for the (Et)^2 contribution to the total value of L^2 for both the spin 1 and spin 1/2 transluminal energy quantum photon models when t is large compared to the period t = 1/f of the photon.</div><div class=""><br class=""></div><div class="">Chip, I also noticed in your calculation for the spin of your electron model in your email below,  you calculate the electron spin as s=r x p where p =E/c = gamma mc is the momentum of the helically moving photon. But we have seen that the transverse component of this total momentum p=E/c of the helically moving photon is only Ptrans = Eo/c = mc  and not E/c = gamma mc. So when this Ptrans=Eo/c is multiplied by <span style="font-family: 'Times New Roman', serif; font-size: 12pt; background-color: rgb(255, 255, 255);" class="">r = (c h)/(4pi E)=</span><span style="font-family: 'Times New Roman', serif; font-size: 16px; background-color: rgb(255, 255, 255);" class="">(c h)/(4pi gamma Eo)</span><span style="font-family: 'Times New Roman', serif; font-size: 12pt; background-color: rgb(255, 255, 255);" class=""> </span> in your model, the result is S = r x Ptrans = h/(4 pi gamma) = hbar/(2 gamma) and not hbar/2 as you state below in your email. And you have apparently left out the contribution of the spin of the helically moving  photon itself in your calculation of the total spin of the helically moving photon model of the electron.</div><div class=""><br class=""></div><div class="">  I hope this is useful.</div><div class="">       Richard</div><div class=""><br class=""></div><div class=""><br class=""></div><div class=""><br class=""></div><div class=""><br class=""></div><div class=""></div></body></html>